Physics > Current Electricity > 5.0 Electromotive force $\left( \xi \right)$

  Current Electricity
    1.0 Introduction
    2.0 Conduction of current in a metal
    3.0 Ohm's law
    4.0 Combination of Resistors
    5.0 Electromotive force $\left( \xi \right)$
    6.0 Heating effect of current
    7.0 Wheatstone bridge
    8.0 Metre Bridge Or Slide wire bridge
    9.0 Potentiometer
    10.0 Electrical devices

5.1 Combination of cells
In series.
If $n$ cells of emf $\left( {{\xi _1},{\xi _2}.....{\xi _n}} \right)$ and internal resistance $\left( {{r_1},{r_2}.....{r_n}} \right)$ are connected in series to external resistance $R$.

Then Equivalent emf, $({\xi _{eq}})$, $$ \Rightarrow {\xi _{eq}} = {\xi _1} + {\xi _2}..... + {\xi _n}$$
Also, equivalent internal resistance $({r_{eq}})$ ,$$ \Rightarrow {r_{eq}} = {r_1} + {r_2} + ..... + {r_n}$$
Total resistance $(R_{eq})$ of the circuit, $$R_{eq}=R + r_{eq}$$
The above circuit becomes as shown.
The current $(I)$ in the circuit, $$I = \frac{{\xi_{eq} }}{{R + r_{eq}}}$$

For $n$ identical cell

Equivalent emf $({\xi _{eq}})$, $$ \Rightarrow {\xi _{eq}} = n\xi $$
Equivalent internal resistance $(r_{eq})$ ,$${r_{eq}} =nr$$
The current $(I)$ in the circuit, $$I = \frac{n{\xi}}{{R + nr}}$$

Special Cases

(i). If $R \gg nr$, then $I = \frac{{n\xi }}{R}$= n times the current due to single cell

(ii).If $R \ll nr$, then $I = \frac{{n\xi }}{{nr}} = \frac{\xi }{r}$ = current given by single cell.

Also,

When the polarity of one cell having emf ${\xi _2}$ and internal resistance ${{r_2}}$ is reversed as shown in the figure.

Then, Equivalent emf $({\xi _{eq}})$, $${\xi _{eq}} = {\xi _1} - {\xi _2}..... + {\xi _n}$$
Also, equivalent internal resistance $({r_{eq}})$ ,$${r_{eq}} = {r_1} + {r_2} + ..... + {r_n}$$


In parallel

$n$ identical cells connected in parallel.

Let there $n$ identical rows each containing a cell of emf $\xi$ and internal resistane $r$ connected with an external resistance $R$ as shown in the figure.

Let ${I_1},{I_2}...{I_n}$ be the current in each row as shown in the figure.

Applying Kirchhoff's junction law at point $A$ we get, $$ \Rightarrow {I_1} + {I_2} + ... + {I_n} = I$$

Applying Kirchhoff's loop law for each loop we get, $$\begin{equation} \begin{aligned} \xi - {I_1}r - IR = 0 \\ \xi - {I_2}r - IR = 0 \\ ..... \\ ..... \\ \xi - {I_n}r - IR = 0 \\\end{aligned} \end{equation} $$

Adding the above equations we get, $$\begin{equation} \begin{aligned} \Rightarrow n\xi - \left( {{I_1} + {I_2} + ... + {I_n}} \right)r - nIR = 0 \\ \Rightarrow n\xi - Ir - nIR = 0 \\ \Rightarrow I = \frac{{n\xi }}{{nR + r}} \\\end{aligned} \end{equation} $$

Special cases

(i) If $R \ll \frac{r}{n}$ , then $I = \frac{{n\xi }}{r}$ = $n$ times current due to single cell.

(ii) If $R \gg \frac{r}{n}$, then $I = \frac{\xi }{R}$, current given by single cell.

Mixed Grouping of cells

Let there $m$ identical rows each containing $n$ cells each of emf $\xi$ and internal resistane $r$ connected with an external resistance $R$ as shown in the figure.

Let ${I_1},{I_2}...{I_n}$ be the current in each row as shown in the figure.

Applying Kirchhoff's junction law at point $A$ we get, $$ \Rightarrow {I_1} + {I_2} + ... + {I_m} = I$$


Applying Kirchhoff's loop law for each loop we get, $$\begin{equation} \begin{aligned} n\xi - {I_1}(nr) - IR = 0 \\ n\xi - {I_2}(nr) - IR = 0 \\ ..... \\ ..... \\ n\xi - {I_m}(nr) - IR = 0 \\\end{aligned} \end{equation} $$

Adding the above equations we get, $$\begin{equation} \begin{aligned} \Rightarrow mn\xi - \left( {{I_1} + {I_2} + ... + {I_m}} \right)(nr)mIR = 0 \\ \Rightarrow mn\xi - Inr - mIR = 0 \\ \Rightarrow I = \frac{{mn\xi }}{{mR + nr}} \\\end{aligned} \end{equation} $$

Question 10. A wire of resistance $4R$ is bent in the form of circle. What is the effective resistance between ends of diameter?


Solution: When the wire of resistance $4R$ is bend in the form of a circle, then the current will flow as shown in the figure.

So, the above situation can be understood as, parallel combination of resistance as showed in the diagram below.

The resistance are in parallel with each other. So the resistance between end $A$ and $B$ is, $$\begin{equation} \begin{aligned} \Rightarrow \frac{1}{{{R_{eq}}}} = \frac{1}{{2R}} + \frac{1}{{2R}} \\ \Rightarrow \frac{1}{{{R_{eq}}}} = \frac{1}{R} \\ \Rightarrow {R_{eq}} = R \\\end{aligned} \end{equation} $$

Question 11. In the given network of resistors. Find the equivalent resistance between point $A$ and $B$.

Solution.

Resistors $AD$ and $DC$ are in series. So, $$\begin{equation} \begin{aligned} \Rightarrow {R_{ADC}} = {R_{AD}} + {R_{DC}} \\ \Rightarrow {R_{ADC}} = (3 + 7)\Omega \\ \Rightarrow {R_{ADC}} = 10\Omega \\\end{aligned} \end{equation} $$
Resistors $R_{ADC}$ and $R_{AC}$ are in parallel. So, $$\begin{equation} \begin{aligned} \Rightarrow \frac{1}{R} = \frac{1}{{{R_{ADC}}}} + \frac{1}{{{R_{AC}}}} \\ \Rightarrow \frac{1}{R} = \frac{1}{{10}} + \frac{1}{{10}} = \frac{2}{{10}} \\ \Rightarrow R = 5\Omega \\\end{aligned} \end{equation} $$
Now, resistors $R$ and $R_{CB}$ are in series. So, $$\begin{equation} \begin{aligned} \Rightarrow R' = R + {R_{CB}} \\ \Rightarrow R' = (5 + 5)\Omega \\ \Rightarrow R' = 10\Omega \\\end{aligned} \end{equation} $$
Finally resistors $R'$ and $R_{AB}$ are in parallel. So, $$\begin{equation} \begin{aligned} \Rightarrow \frac{1}{R_{AB}} = \frac{1}{R'} + \frac{1}{{{R_{AB}}}} \\ \Rightarrow \frac{1}{R_{AB}} = \frac{1}{{10}} + \frac{1}{{10}} = \frac{2}{{10}} \\ \Rightarrow {R_{AB}} = 5\Omega \\\end{aligned} \end{equation} $$

Question 12. Calculate total current flowing through the circuit.

Solution. The circuit diagram can be redrawn as shown in figure.

Now, the equivalent resistance can be very calculated. $$R_{eq}=2\Omega$$

Thus, Current flowing through circuit is $$I = \frac{V}{R} = \frac{{10V}}{{2\Omega }} = 5A$$

Question 13. In the circuit shown, calculate the effective resistance.

Solution. Effective resistance between $E$ and $F$ ,
Resistance ${2\Omega }$, ${4\Omega }$ and ${2\Omega }$ are in series and their equivalent is in parallel with ${8\Omega }$

Effective resistance is $$R_{EF} = \frac{{8 \times 8}}{{8 + 8}} = 4\Omega $$

Similarly, effective resistance between $C$ and $D$ is $$R_{CD} = \frac{{8 \times 8}}{{8 + 8}} = 4\Omega $$

Then finally resistance ${3\Omega }$, ${4\Omega }$ and ${2\Omega }$ are in series.

Equivalent resistance of the circuit $$R_{AB} = 3 + 4 + 2 = 9\Omega $$

Question 14. A cell of emf $4V$ and internal resistance 0.2$\Omega $ is connected to a 7.8$\Omega $ external resistance. What will be the potential difference across the terminals of cell?

Solution.

Given $\xi = 4V$ , $r =0.2\Omega $ , $R=7.8\Omega $

Current, $$I = \frac{\xi }{{R + r}} = \frac{4}{{7.8 + 0.2}} = 0.5A$$
Potential difference across terminals $AB$ of cell, $$V_{AB} = IR = 0.5 \times 7.8 = 3.9V$$
or
Potential difference across terminals $AB$ of cell, $$V_{AB} = \xi - Ir = 4-0.2 \times 0.5 = 3.9V$$
Improve your JEE MAINS score
10 Mock Test
Increase JEE score
by 20 marks
Detailed Explanation results in better understanding
Exclusively for
JEE MAINS and ADVANCED
9 out of 10 got
selected in JEE MAINS
Lets start preparing
DIFFICULTY IN UNDERSTANDING CONCEPTS?
TAKE HELP FROM THINKMERIT DETAILED EXPLANATION..!!!
9 OUT OF 10 STUDENTS UNDERSTOOD